Partículas idénticas parecen reducir la probabilidad

Esta pregunta básicamente tiene dos partes muy relacionadas. Esto surgió en el contexto de tratar de verificar algo que dijo mi profesor hace un tiempo: que si las funciones de onda para dos partículas idénticas están bien separadas (es decir, si tienen un pico muy alto y los picos están separados macroscópicamente), entonces puedes modelarlos como partículas distinguibles. Razonó que cuando intercambiamos las funciones de onda, el término que obtenemos del intercambio es muy pequeño y puede ignorarse. Entonces, tomar la norma de la función de onda simetrizada se reduce a tomar la norma de la amplitud no trivial en la simetrización. Esta será la amplitud que obtendría al modelar las partículas como distinguibles.

Sin embargo, cuando hago este procedimiento, la normalización del procedimiento de simetrización me fastidia. No estoy seguro de dónde me estoy equivocando.

Supongamos que tengo dos bosones. yo se que uno esta en el estado | ψ 1 y uno está en el estado | ψ 2 .

El estado simetrizado es entonces

1 2 ( | ψ 1 ψ 2 + | ψ 2 ψ 1 )

Supongamos que las funciones de onda para | ψ 1 y | ψ 2 tienen un punto máximo en un lugar separado o tienen un soporte que no se superpone.

Entonces, si miro X 1 , X 2 , con X 1 apoyar ( ψ 1 ( ) ) y X 2 apoyar ( ψ 2 ( ) ) , entonces no será la probabilidad de que observe una partícula cerca X 1 y otro cerca X 2 ser

1 2 | ψ 1 ( X 1 ) | 2 | ψ 2 ( X 2 ) | 2

que es la mitad de lo que sería para las partículas distinguibles? Este resultado se siente mal. Pensé que tal vez algo andaba mal con la normalización, pero no parece ser que lo haya.

Finalmente, una pregunta de física en la lista HNQ que no se puede responder con un libro de texto escolar o un artículo de Wikipedia. +1.

Respuestas (3)

La gente suele decir que "partículas cuánticas indistinguibles que están muy separadas se comportan como partículas distinguibles", pero esto es un poco engañoso. Sería más exacto decir "partículas cuánticas indistinguibles que están muy separadas se comportan como partículas clásicas indistinguibles ". La diferencia es sutil pero importante.

El problema está en tu oración "si miro X 1 , X 2 , con X 1 apoyar ( ψ 1 ( ) ) y X 2 apoyar ( ψ 2 ( ) ) ...". No hay ningún experimento posible que diga "He detectado partículas X 1 en el lugar X " - solo experimentos que dicen "He detectado una partícula en la ubicación X ." Entonces, la amplitud de probabilidad de que mida una partícula en el punto X y otra partícula en el punto y es

X , y | 1 2 ( | ψ 1 ψ 2 + | ψ 2 ψ 1 ) = 1 2 ( ψ 1 ( X ) ψ 2 ( y ) + ψ 2 ( X ) ψ 1 ( y ) )

y la probabilidad real es el cuadrado normal

PAG ( X , y ) = 1 2 ( | ψ 1 ( X ) | 2 | ψ 2 ( y ) | 2 + | ψ 2 ( X ) | 2 | ψ 1 ( y ) | 2 ) ,

donde los términos cruzados son todos cero porque los estados | ψ 1 y | ψ 2 son ortogonales. Este es de hecho el resultado de las partículas indistinguibles clásicas , y PAG ( X , y )   d X d y = 1 como debería (a diferencia de su expresión).

Las únicas cosas que difieren del caso cuántico genérico son (a) solo puede usar el factor de normalización de 1 / 2 y (b) cuando eleva al cuadrado la amplitud a una densidad de probabilidad real, puede ignorar los términos cruzados.

Si desea trabajar de forma clásica desde el principio, entonces no debe simetrizar el ket en absoluto, sino trabajar con el ket | ψ 1 ψ 2 en cambio. Entonces su problema de normalización no aparece.

Esto no explica la pregunta. Resuelve el problema al redefinir lo que PAG ( X , y ) significa. Para obtener una probabilidad (ignorando el hecho de que en realidad es una densidad) de encontrar una partícula en X 1 y otro en X 2 uno tendría que sumar manualmente PAG ( X 1 , X 2 ) + PAG ( X 2 , X 1 ) (siendo los dos términos iguales). ¿Qué evento hace entonces PAG ( X , y ) solo describir como probabilidad ?
(a) Esto no tenía la intención de abordar su respuesta, lo siento, simplemente no quería decir "densidad" como tres veces, así que lo dije una vez con un descargo de responsabilidad. (b) Por supuesto que no entiendo la física cuántica. (c) No lo es y ese es exactamente el corazón de mi comentario. Es la mitad de la probabilidad en cuestión porque para obtener la probabilidad tendrías que sumar PAG ( X , y ) con PAG ( y , X ) .
(Esto último no es una cuestión de mecánica cuántica. En un ejemplo mínimo, dos partículas indistinguibles tienen dos estados posibles, a y b , cada. Entonces todo lo que podemos saber es, por ejemplo, "están en diferentes estados". Si requerimos eso X , y PAG ( X , y ) = 1 , esto significa para PAG eso PAG ( a , b ) = 1 2 , PAG ( b , a ) = 1 2 y PAG ( a , a ) = PAG ( b , b ) = 0 . Sin embargo, la probabilidad de encontrar una partícula en a y uno en b es uno. Eso no es PAG ( X , y ) para cualquier X , y .)
Pero son la mitad de eso. De lo contrario, su integral tendría que evitar la doble suma imponiendo un límite X y o algo. Por cierto, eso | a | b H es básicamente lo que dice mi propia respuesta, y el problema desaparece si eso se resuelve primero.
Ah, y "uno en..." significaba números de ocupación, como en el OP. No los considero distintos.
No estoy confundido; solo trato de probar mi punto. ¿Podría comentar sobre mi ejemplo mínimo? La probabilidad de encontrar el estado | norte a = 1 , norte b = 1 es un tiempo PAG ( a , b ) = PAG ( b , a ) son 1 2 y no uno Eso es, según tengo entendido, problema del OP.
@TheVee Sí, tienes razón y yo estaba equivocado. Una densidad de probabilidad de partículas múltiples PAG ( X , y , z . . . ) es siempre simétrico bajo todas las permutaciones (incluso para fermiones). La normalización estándar asume que está integrando todo el espacio de configuración sin restricciones, por lo que necesita multiplicar PAG por norte ! para obtener la probabilidad física para dar cuenta de la indistinguibilidad clásica de las partículas (es decir, que cada configuración física es en realidad una clase de equivalencia de norte ! diferentes puntos en el espacio de configuración, o si quiere ser elegante, el espacio de estado físico es en realidad ...
Espacio de configuración de @TheVee modificado por el grupo de permutaciones de partículas. Si el espacio de configuración es ordenable, entonces esto es básicamente solo norte ! copias de la cuña X 1 < X 2 < . Para los bosones, aunque no para los fermiones, hay puntos degenerados donde múltiples partículas están en la misma ubicación y el factor combinatorio cambia, pero son de medida cero y no afectan las integrales de probabilidad reales, por lo que podemos ignorar esa sutileza).

En lugar de escribir:

1 2 ( | ψ 1 ψ 2 + | ψ 2 ψ 1 )

Será más instructivo ver esto como

Ψ ( X 1 , X 2 ) = 1 2 ( ψ 1 ( X 1 ) ψ 2 ( X 2 ) + ψ 1 ( X 2 ) ψ 2 ( X 1 ) )

Así cuando encuentres | Ψ | 2 , obtienes tres (técnicamente 4) términos:

1 2 ( | ψ 1 ( X 1 ) ψ 2 ( X 2 ) | 2 + ( términos de interacción despreciables ) + | ψ 1 ( X 2 ) ψ 2 ( X 1 ) | 2 )

Pero para partículas indistinguibles, ψ 1 ( X j ) = ψ 1 ( X i ) . Aunque los términos de interacción desaparecen a gran distancia, este hecho es cierto independientemente de la distancia entre X i y X j !. Explotando ese hecho e ignorando los términos de interacción, obtenemos:

1 2 ( 2 | ψ 1 ( X 1 ) ψ 2 ( X 2 ) | 2 )

Y entonces, el resultado termina siendo el mismo que el de las partículas distinguibles, pero solo toma una ruta diferente para llegar allí.

Edito: recuerda que es Ψ y no ψ que debe ser simétrico/antisimétrico para bosones/fermiones respectivamente bajo el intercambio de partículas X 1 y X 2 .

ψ 1 ( X j ) no puede ser trivialmente igual a ψ 1 ( X i ) en la notación OP. Sólo uno de los dos puntos se encuentra en el soporte de esa función.
@TheVee comentario tonto, sabía lo que quería decir, pero lo que escribió y tú también.
¿Hablas en serio? Esta es una discrepancia flagrante. No se trata de lo que quiso decir. Para una función ψ 1 con apoyo centrado alrededor de 1 y ψ 2 centrado alrededor de 2 y puntos X 1 = 1 y X 2 = 2 ¿Cómo podrías reclamar ψ 1 ( X 1 ) = ψ 1 ( X 2 ) ?
@TheVee uh... se refiere al índice de partículas y no a valores x arbitrarios.

Debe usar la simetrización bosónica de manera consistente (también con el estado medido) y decir que la probabilidad de detectar una partícula en | ϕ 1 y uno en | ϕ 2 , donde ϕ 1 | ϕ 2 = 0 , es la magnitud al cuadrado del producto escalar entre

| Ψ = 1 2 ( | ψ 1 | ψ 2 + | ψ 2 | ψ 1 )
y
| Φ = 1 2 ( | ϕ 1 | ϕ 2 + | ϕ 2 | ϕ 1 ) .
Es decir, después de la expansión, el cuadrado magnético de
Φ | Ψ = ϕ 1 | ψ 1 ϕ 2 | ψ 2 + ϕ 2 | ψ 1 ϕ 1 | ψ 2
en lugar de
( ϕ 1 | ϕ 2 | ) | Ψ = 1 2 ( ϕ 1 | ψ 1 ϕ 2 | ψ 2 + ϕ 2 | ψ 1 ϕ 1 | ψ 2 )
que usaste

Si dobla esta fórmula para usarla con estados propios de posición generalizados separados espacialmente, obtendrá el resultado correcto.